16
$\begingroup$

Let $n\geq 0$ be an integer and let $J_n=J_n(r)$ denote the usual Bessel function (of the first kind) of order $n$ i.e. one of the solutions to Bessel's differential equation

$$r^2\frac{d^2y}{dr^2}+r\frac{dy}{dr}+(r^2-n^2)y=0.$$

I am interested in the quantities

$$\beta_n:=\int_0^\infty J_n(r)^2 J_0(r)^2 dr$$ which arise naturally in the study of certain classes of oscillatory integrals. In particular, I would like to explore the convexity of the sequence $\{\beta_n\}$. My question is the following: is it true that

$$\beta_n<\frac{\beta_{n-1}+\beta_{n+1}}{2}$$ for every $n\geq 1$? This inequality seems to be numerically verifiable (Mathematica) for small values of $n$. To prove it, I was trying to use the so-called ''Neumann's formula'' (see e.g. Watson's treatise (1966), p.32)

$$J_n(r)^2=\frac{1}{\pi}\int_0^\pi J_{2n}(2r\sin\theta)d\theta,$$ together with the well-known recurrence

$$J_{n-1}(r)-J_{n+1}(r)=2 \frac{d J_n}{dr}(r)$$ and some integration by parts. Unfortunately I wasn't able to make this approach work.

Ideas or/and references are welcome, thank you very much in advance.

$\endgroup$

2 Answers 2

2
$\begingroup$

I fully suspect the answer to your question is yes. The reason is as follows. \begin{eqnarray} \beta_{n-1} + \beta_{n+1} &=& \int [J_{n+1}^2 + J_{n-1}^2] J_0^2 \nonumber \\ &=& \int [(J_{n+1} + J_{n-1})^2 - 2 J_{n+1} J_{n-1}] J_0^2 \nonumber \\ &=& \int [\frac{4 n^2}{r^2} J_n^2 - 2 J_{n+1} J_{n-1}] J_0^2 \end{eqnarray} where we we use the recurrence relation $2 \frac{n}{2} J_n = J_{n-1} + J_{n+1}$ to get the last inequality. Notice that since $$\int 2J_{n+1} J_{n-1}J_0^2 \leq \beta_{n+1} + \beta_{n-1}$$ we have that $$ \beta_n + \int (\frac{n^2}{r^2} - 1) J_n^2 J_0^2 \leq \frac{\beta_{n-1} + \beta_{n+1}}{2} $$ You now just need that one equation to be non-negative to get the answer you want. This quantity should be non-negative by the simple fact that $\frac{n^2}{r^2} - 1$ is positive near $r=0$ and this is where the Bessel functions have most of their mass. Further, $\frac{n^2}{r^2} - 1$ is negative where the Bessel function is small. I'm not sure how to get this rigorously though.

I hope this helps.

$\endgroup$
4
  • $\begingroup$ Thank you! Your heuristics are insightful but I think you're missing one point: the function $J_n(r)$ starts oscillating only around $r\sim n$, until then it looks like a convex function with absolute value very close to 0. And in fact I just checked (numerically) that the integrals $\int (n^2/r^2-1)J_n^2 J_0^2$ are negative for small values of $n$. I also suspect that my original question should have an affirmative answer, but something rather subtle seems to be going on... Any further thoughts? Thanks again. $\endgroup$
    – user17240
    Apr 12, 2014 at 15:23
  • 2
    $\begingroup$ That's interesting. I didn't think the estimate would have been that damaging. The only place where we estimated was to replace $2ab$ with $a^2 + b^2$. If we don't estimate, then we get $2ab = a^2 + b^2 - (a - b)^2$. Setting $a = J_{n+1}$ and $b=J_{n-1}$ and using the recurrence relation $2 J_n' = J_{n-1} - J_{n+1}$ we get $$\beta_n + \int\{(\frac{n^2}{r^2} - 1)J_n^2 + J_n'^2\}J_0^2 = \frac{\beta_{n-1} + \beta_{n+1}}{2}$$ It seems interesting that one gets that $(\frac{n^2}{r^2} - 1)J_n^2 + J_n'^2$ turns out to be non-negative in an averaged sense. I'll keep thinking about this. $\endgroup$
    – k3thomps
    Apr 12, 2014 at 20:19
  • $\begingroup$ I don't think a 2 is missing. I re-did the calculation and what I had above looks correct. $\endgroup$
    – k3thomps
    May 21, 2014 at 18:03
  • $\begingroup$ @Wurlitzer and then you have to divide by 4. $\endgroup$
    – k3thomps
    May 22, 2014 at 12:43
2
$\begingroup$

This is not an answer, just a comment following k3thomps computations in remarks, the question is equivalent to, it seems: why is $$ \int_0^\infty \left(\left(\frac{n^2}{x^2}-1\right) J_n^2 +(J_n^\prime)^2\right)J_0^2\text{d}x> 0 \,? $$ Below is an alternative formulation of the same question (I can't make it work, but here it is anyway). We have, by definition, $$J_n^{\prime\prime}+\frac{1}{x}J_n ^\prime-\left(\frac{n^2}{x^2}-1\right) J_n=0$$ Integrating against $J_nJ_0^2$, $n\geq1$, we obtain \begin{eqnarray*} &&\int_0^\infty \left(\left(\frac{n^2}{x^2}-1\right) J_n^2 + (J_n^\prime)^2\right) J_0^2 \text{d}x\\ &=& \frac{1}{2}\int_0^\infty -(J_n^2)^\prime(J_0^2)^\prime +\frac{1}{x}(J_n^2)^\prime J_0^2\text{d}x\\ &=& \frac{1}{2}\int_0^\infty J_n^2\left((J_0^2)^{\prime\prime} -\left(\frac{1}{x}J_0^2\right)^\prime \right)\text{d}x. \end{eqnarray*}

This is puzzling, because $$(J_0^2)^{\prime\prime} -\left(\frac{1}{x}J_0^2\right)^\prime$$ oscillates on $(0,\infty)$.

Nevertheless, using the idea suggested in the original post, let $Q_{n}\left(r\right)=J_{n}^{2}\left(r\right).$ We have \begin{align*} \int_{0}^{\infty}\left(\frac{1}{r}J_{n}^{\prime}J_{n}+\left(J_{n}J_{n}^{\prime}\left(r\right)\right)^{\prime}\right)J_{0}^{2}\text{d}r & =\frac{1}{2}\int_{0}^{\infty}\left(\frac{1}{r}Q_{n}^{\prime}+Q_{n}^{\prime\prime}\right)Q_{0}\text{d}r\\ & =\frac{1}{2}\int_{0}^{\infty}\left(-\frac{1}{r}Q_{0}+Q_{0}^{\prime}\right)^{\prime}Q_{n}\text{d}r. \end{align*} Now, \begin{align*} Q_{0}\left(r\right) & =\frac{1}{\pi}\int_{0}^{\pi}J_{0}\left(2r\sin\psi\right)\text{d}{\psi},\\ Q_{n}\left(r\right) & =\frac{1}{\pi}\int_{0}^{\pi}J_{0}\left(2r\sin\varphi\right)\cos\left(2n\varphi\right)\text{d}{\varphi}, \end{align*} and \begin{align*} Q_{n}^{\prime} & =\frac{1}{\pi}\int_{0}^{\pi}J_{0}^{\prime}\left(2r\sin\varphi\right)2\sin\varphi\cos\left(2n\varphi\right)\text{d}{\varphi}\\ Q_{n}^{\prime\prime} & =\frac{1}{\pi}\int_{0}^{\pi}J_{0}^{\prime\prime}\left(2r\sin\varphi\right)\left(2\sin\varphi\right)^{2}\cos\left(2n\varphi\right)\text{d}{\varphi} \end{align*} Recall that $$ rJ_{0}^{\prime\prime}(r)+J_{0}^{\prime}(r)+rJ_{0}=0, $$ thus \begin{align*} Q_{n}^{\prime\prime}= & \frac{1}{r}\frac{1}{\pi}\int_{0}^{\pi}J_{0}^{\prime\prime}\left(2r\sin\varphi\right)\left(2r\sin\varphi\right)\left(2\sin\varphi\right)\cos\left(2n\varphi\right)\text{d}{\varphi}\\ = & -\frac{1}{r}\frac{1}{\pi}\int_{0}^{\pi}J_{0}^{\prime}\left(2r\sin\varphi\right)\left(2\sin\varphi\right)\cos\left(2n\varphi\right)\text{d}{\varphi}\\ & -\frac{1}{\pi}\int_{0}^{\pi}J_{0}\left(2r\sin\varphi\right)\left(2\sin\varphi\right)\cos\left(2n\varphi\right)\text{d}{\varphi},\\ \left(\frac{1}{r}Q_{n}^{\prime}+Q_{n}^{\prime\prime}\right) & =-\frac{1}{\pi}\int_{0}^{\pi}J_{0}\left(2r\sin\varphi\right)\left(2\sin\varphi\right)\cos\left(2n\varphi\right)\text{d}{\varphi},\\ \left(\frac{1}{r}Q_{n}^{\prime}+Q_{n}^{\prime\prime}\right)Q_{0} & =-\frac{1}{\pi}\int_{0}^{\pi}J_{0}\left(2r\sin\varphi\right)2\sin\varphi\cos\left(2n\varphi\right)\text{d}{\varphi} \times \\ &\frac{1}{\pi}\int_{0}^{\pi}J_{0}\left(2r\sin\theta\right)d\theta,\\ \frac{1}{2}\int_{0}^{\infty}\left(\frac{1}{r}Q_{n}^{\prime}+Q_{n}^{\prime\prime}\right)Q_{0}\text{d}r & =-\frac{1}{\pi^{2}}\int_{0}^{\pi}\text{d}\theta\int_{0}^{\pi}2\sin\varphi\cos\left(2n\varphi\right)\text{d}{\varphi} \times \\ &\int_{0}^{\infty}J_{0}\left(2t\sin\theta\right)J_{0}\left(2t\sin\varphi\right)\text{d}t \end{align*} Now, it appears that for $\alpha,\beta>0$ $\int_{0}^\infty J_{0}(\alpha r)J_{0}(\beta r) \text{d} r $ is a known quantity, a so-called complete elliptic integral of the first type named $K$, given by $$ \int_{0}^\infty J_{0}(\alpha r)J_{0}(\beta r) \text{d} r = \frac{2}{\pi \max(\alpha,\beta)} K \left(\frac{\min(\alpha,\beta)}{\max(\alpha,\beta)}\right) $$

Now, more DLMF or algebraic manipulations to sort out the subsequent integrals on triangles..to be continued (one day).

$\endgroup$

Your Answer

By clicking “Post Your Answer”, you agree to our terms of service and acknowledge you have read our privacy policy.

Not the answer you're looking for? Browse other questions tagged or ask your own question.